wu :: forums
« wu :: forums - limit of cosine to the nth at n »

Welcome, Guest. Please Login or Register.
Apr 19th, 2024, 8:54pm

RIDDLES SITE WRITE MATH! Home Home Help Help Search Search Members Members Login Login Register Register
   wu :: forums
   riddles
   putnam exam (pure math)
(Moderators: william wu, Eigenray, Icarus, Grimbal, towr, SMQ)
   limit of cosine to the nth at n
« Previous topic | Next topic »
Pages: 1  Reply Reply Notify of replies Notify of replies Send Topic Send Topic Print Print
   Author  Topic: limit of cosine to the nth at n  (Read 994 times)
william wu
wu::riddles Administrator
*****





   
WWW

Gender: male
Posts: 1291
limit of cosine to the nth at n  
« on: Jan 19th, 2004, 12:13am »
Quote Quote Modify Modify

Does the following limit exist?
 
lim n[to][infty] cosn(n), where n is a natural number

 
If so, what's the limit? If not, why not?
« Last Edit: Jan 19th, 2004, 12:15am by william wu » IP Logged


[ wu ] : http://wuriddles.com / http://forums.wuriddles.com
towr
wu::riddles Moderator
Uberpuzzler
*****



Some people are average, some are just mean.

   


Gender: male
Posts: 13730
Re: limit of cosine to the nth at n  
« Reply #1 on: Jan 19th, 2004, 1:23am »
Quote Quote Modify Modify

::If the limit exists it would probably be 0, because |cos(n)| < 1 for all n. It can however get arbitrarily close to 1, so I'm not sure. I'd guess the first, but since that would seem obvious perhaps it's the second..::
IP Logged

Wikipedia, Google, Mathworld, Integer sequence DB
Sir Col
Uberpuzzler
*****




impudens simia et macrologus profundus fabulae

   
WWW

Gender: male
Posts: 1825
Re: limit of cosine to the nth at n  
« Reply #2 on: Jan 19th, 2004, 7:13am »
Quote Quote Modify Modify

::
If |a|<1, |a.ak|<|ak|, therefore an[to]0 as n[to][infty].
 
As towr pointed out, if n is a natural number, then it cannot be a multiple of pi, so |cos(n)|<1. Hence cosn(n)[to]0 as n[to][infty].
 
I suppose an interesting question would be to consider the limit of cosx(x), as x[to][infty].
 
I suspect that no limit exists, simply because, cos(x)=[pm]1, for x=k[pi]. It even becomes difficult (for me) to imagine the function being continuous for very large values of x.  Huh
::
IP Logged

mathschallenge.net / projecteuler.net
towr
wu::riddles Moderator
Uberpuzzler
*****



Some people are average, some are just mean.

   


Gender: male
Posts: 13730
Re: limit of cosine to the nth at n  
« Reply #3 on: Jan 19th, 2004, 7:49am »
Quote Quote Modify Modify

I think you are forgetting one thing, or at the very least omit to proof it isn't the case.
|cos(n)| could go faster to 1 than the power drives it down to zero.  
For instance like (1-2^-n)^n; (1-2^-n) is allways smaller than 1, but goes to 1 in the limit faster than the power can drive it back down.  
In the case of cos(n) you can get increasingly better approximations of multiples of 2[pi] (f.i. just take round(2m[pi]) )
 
IP Logged

Wikipedia, Google, Mathworld, Integer sequence DB
Barukh
Uberpuzzler
*****






   


Gender: male
Posts: 2276
Re: limit of cosine to the nth at n  
« Reply #4 on: Jan 19th, 2004, 8:13am »
Quote Quote Modify Modify

Sir Col, I think the original question is not so simple… It’s very important to recall what the definition of the limit of the sequence is, so let me give it here: The sequence {an} has the limit A as n [to][infty] if for every positive number [epsilon] there exists an integer N (depending on [epsilon]) such that |a - an| < [epsilon] for all n [ge] N.
 
The words “for all” are essential here. Now, if I give you an [epsilon], say, 2-10, could you find such an N? towr has indicated that this may be not the case since cos(n) may be arbitrarily close to 1. We know that cos(n) = cos(n mod 2[pi]), and the argument of the latter – as was shown on another thread – is uniformly distributed in (0, 1).
IP Logged
Eigenray
wu::riddles Moderator
Uberpuzzler
*****






   


Gender: male
Posts: 1948
Re: limit of cosine to the nth at n  
« Reply #5 on: Jan 19th, 2004, 12:50pm »
Quote Quote Modify Modify

By Dirichlet's Approximation Theorem, for any positive integer N, there are integers h,k, 0<k<=N such that |k[pi]-h| < 1/N.
Thus 1 > |cos(h)| > cos(1/N), and also, 0 < h < k[pi] + 1/N < 5N.
Thus |cosh(h)| > cos5N(1/N) ([to]1).
 
The next claim is that h may be chosen arbitrarily large:
To see this, fix M > 0.  Then [epsilon] := min { |k[pi]-h| : 0 < h < M, 0 < k < (h+1)/[pi] } > 0, so by picking N > 1/[epsilon], the corresponding h must be larger than M.
Thus there is a sequence Ni[to][infty], hi[to][infty], such that
|cosh_i(hi)| > cos5N_i(1/Ni).
Taking limits shows that limsup |cosn(n)| = 1.
(In particular, the sequence cosn(n) gets arbitrarily close to 1 infinitely often.)
 
A similar argument, however, show that liminf |cosn(n)| = 0, thus lim cosn(n) does not exist.
IP Logged
Sir Col
Uberpuzzler
*****




impudens simia et macrologus profundus fabulae

   
WWW

Gender: male
Posts: 1825
Re: limit of cosine to the nth at n  
« Reply #6 on: Jan 19th, 2004, 1:31pm »
Quote Quote Modify Modify

Thanks for correcting me, guys; I see what you're both driving at. Just when we thought we'd found n0 such that |cosn(n)|<[epsilon], for all n[ge]n0, some later value of n is above it.
 
I can see that this problem required tools that are outside of my limited mathematical repertoire.  Smiley
« Last Edit: Jan 19th, 2004, 1:34pm by Sir Col » IP Logged

mathschallenge.net / projecteuler.net
Icarus
wu::riddles Moderator
Uberpuzzler
*****



Boldly going where even angels fear to tread.

   


Gender: male
Posts: 4863
Re: limit of cosine to the nth at n  
« Reply #7 on: Jan 19th, 2004, 5:50pm »
Quote Quote Modify Modify

on Jan 19th, 2004, 1:31pm, Sir Col wrote:
I can see that this problem required tools that are outside of my limited mathematical repertoire.  Smiley

 
Probably not, though you did make that one oversight. It is a fairly common mistake in this sort of situation.
 
But the Dirichlet Approximation Theorem, as hi-falutin as the name sounds, is nothing more than the observation that the rational numbers are dense in the Reals (i.e., they are arbitrarily close to every real number). The more exacting version that EigenRay develops is just noting that restricting the rational to denominators greater than an arbitrary fixed N still leaves a dense set. Both of these are things I believe you are already familiar with, as you have discussed similar things in the past.
IP Logged

"Pi goes on and on and on ...
And e is just as cursed.
I wonder: Which is larger
When their digits are reversed? " - Anonymous
Pages: 1  Reply Reply Notify of replies Notify of replies Send Topic Send Topic Print Print

« Previous topic | Next topic »

Powered by YaBB 1 Gold - SP 1.4!
Forum software copyright © 2000-2004 Yet another Bulletin Board